Newton's Sum is too slow!

Algebra Level 5

x 3 + 2 x 2 + 3 x + 1 = 0 \Large{x^3 + 2x^2 + 3x + 1 = 0}

Let p , q , r p,q,r be three distinct complex numbers that satisfies the above cubic equation. Evaluate the last three digits of:

p 81 + q 81 + r 81 . \large{p^{81} + q^{81} + r^{81} } .


The answer is 148.

This section requires Javascript.
You are seeing this because something didn't load right. We suggest you, (a) try refreshing the page, (b) enabling javascript if it is disabled on your browser and, finally, (c) loading the non-javascript version of this page . We're sorry about the hassle.

3 solutions

Pi Han Goh
Apr 5, 2014

Note that 81 = 3 4 81 = 3^4 .

Let f n ( x ) f_n (x) and g n ( x ) = x 3 f n ( 1 x ) g_n (x) = x^3 f_n \left ( \frac {1}{x} \right ) be 3 rd 3^{\text{rd}} degree polynomials with integer coefficient such that f n ( x ) f_n (x) has zeros p 3 n , q 3 n , r 3 n \large p^{3^n}, q^{3^n}, r^{3^n} while g n ( x ) g_n (x) has zeros p 3 n , q 3 n , r 3 n \large p^{-3^n}, q^{-3^n}, r^{-3^n} , where n = 0 , 1 , 2 , 3 n=0,1,2,3

So f 0 ( x ) = x 3 + 2 x 2 + 3 x + 1 f_0 (x) = x^3 + 2x^2 + 3x + 1 . By Vieta's formula, p + q + r = 2 , p q + p r + q r = 3 , p q r = 1 p+q+r=-2, pq+pr+qr=3, pqr=-1 . Apply the algebraic identity

p 3 + q 3 + r 3 3 p q r = ( p + q + r ) ( ( p + q + r ) 2 3 ( p q + p r + q r ) ) p 3 + q 3 + r 3 = 7 p^3 + q^3 + r^3 - 3pqr = (p+q+r) \left ( (p+q+r)^2 - 3(pq+pr+qr) \right ) \Rightarrow p^3 + q^3 + r^3 = 7

And g 0 ( x ) = x 3 + 3 x 2 + 2 x + 1 1 p + 1 q + 1 r = 3 , 1 p q + 1 p r + 1 q r = 2 g_0 (x) = x^3 + 3x^2 + 2x + 1 \Rightarrow \frac {1}{p} + \frac {1}{q} + \frac {1}{r} = -3, \Rightarrow \frac {1}{pq} + \frac {1}{pr} + \frac {1}{qr} = 2

( 1 p ) 3 + ( 1 q ) 3 + ( 1 r ) 3 3 p q r = ( 1 p + 1 q + 1 r ) ( ( 1 p + 1 q + 1 r ) 2 3 ( 1 p q + 1 p r + 1 q r ) ) \left ( \frac {1}{p} \right )^3 + \left ( \frac {1}{q} \right )^3 + \left ( \frac {1}{r} \right )^3 - \frac {3}{pqr} = \left ( \frac {1}{p} + \frac {1}{q} + \frac {1}{r} \right ) \left ( \left ( \frac {1}{p} + \frac {1}{q} + \frac {1}{r} \right )^2 - 3\left ( \frac {1}{pq} + \frac {1}{pr} + \frac {1}{qr} \right ) \right )

1 p 3 + 1 q 3 + 1 r 3 = 12 p 3 q 3 + p 3 r 3 + q 3 r 3 ( p q r ) 3 = 12 p 3 q 3 + p 3 r 3 + q 3 r 3 = 12 \large \Rightarrow \frac {1}{p^3} + \frac {1}{q^3} + \frac {1}{r^3} = -12 \Rightarrow \frac {p^3 q^3 + p^3 r^3 + q^3 r^3}{(pqr)^3} = -12 \Rightarrow p^3 q^3 + p^3 r^3 + q^3 r^3 = 12

f 1 ( x ) = x 3 7 x 2 + 12 x + 1 g 1 ( x ) = x 3 + 12 x 2 7 x + 1 f_1 (x) = x^3 -7x^2 +12x + 1 \Rightarrow g_1 (x) = x^3 + 12x^2 - 7x + 1

Apply the same method above:

p 9 + q 9 + r 9 3 ( p q r ) 3 = ( p 3 + q 3 + r 3 ) ( ( p 3 + q 3 + r 3 ) 2 3 ( p 3 q 3 + p 3 r 3 + q 3 r 3 ) ) p^9 + q^9 + r^9 - 3(pqr)^3 = (p^3+q^3+r^3) \left ( (p^3+q^3+r^3)^2 - 3(p^3 q^3 +p^3 r^3 +q^3 r^3) \right )

p 9 + q 9 + r 9 = 88 \Rightarrow p^9 + q^9 + r^9 = 88

( 1 p 3 ) 3 + ( 1 q 3 ) 3 + ( 1 r 3 ) 3 3 ( p q r ) 3 = ( 1 p 3 + 1 q 3 + 1 r 3 ) ( ( 1 p 3 + 1 q 3 + 1 r 3 ) 2 3 ( 1 p 3 q 3 + 1 p 3 r 3 + 1 q 3 r 3 ) ) \left ( \frac {1}{p^3} \right )^3 + \left ( \frac {1}{q^3} \right )^3 + \left ( \frac {1}{r^3} \right )^3 - \frac {3}{(pqr)^3} = \left ( \frac {1}{p^3} + \frac {1}{q^3} + \frac {1}{r^3} \right ) \left ( \left ( \frac {1}{p^3} + \frac {1}{q^3} + \frac {1}{r^3} \right )^2 - 3\left ( \frac {1}{p^3 q^3 } + \frac {1}{p^3 r^3} + \frac {1}{q^3 r^3} \right ) \right )

1 p 9 + 1 q 9 + 1 r 9 = 1983 p 9 q 9 + p 9 r 9 + q 9 r 9 = 1983 \large \Rightarrow \frac {1}{p^9} + \frac {1}{q^9} + \frac {1}{r^9} = -1983 \Rightarrow p^9 q^9 + p^9 r^9 + q^9 r^9 =1983

f 2 ( x ) = x 3 88 x 2 + 1983 x + 1 g 2 ( x ) = x 3 + 1983 x 2 88 x + 1 f_2 (x) = x^3 - 88x^2 +1983x + 1 \Rightarrow g_2 (x) = x^3 + 1983x^2 - 88x + 1

Repeat:

p 27 + q 27 + r 27 3 ( p q r ) 9 = ( p 9 + q 9 + r 9 ) ( ( p 9 + q 9 + r 9 ) 2 3 ( p 9 q 9 + p 9 r 9 + q 9 r 9 ) ) p^{27} + q^{27} + r^{27} - 3(pqr)^9 = (p^9+q^9+r^9) \left ( (p^9+q^9+r^9)^2 - 3(p^9 q^9 +p^9 r^9 +q^9 r^9) \right )

p 27 + q 27 + r 27 = 157957 \Rightarrow p^{27} + q^{27} + r^{27} = 157957

( 1 p 9 ) 3 + ( 1 q 9 ) 3 + ( 1 r 9 ) 3 3 ( p q r ) 9 = ( 1 p 9 + 1 q 9 + 1 r 9 ) ( ( 1 p 9 + 1 q 9 + 1 r 9 ) 2 3 ( 1 p 9 q 9 + 1 p 9 r 9 + 1 q 9 r 9 ) ) \left ( \frac {1}{p^9} \right )^3 + \left ( \frac {1}{q^9} \right )^3 + \left ( \frac {1}{r^9} \right )^3 - \frac {3}{(pqr)^9} = \left ( \frac {1}{p^9} + \frac {1}{q^9} + \frac {1}{r^9} \right ) \left ( \left ( \frac {1}{p^9} + \frac {1}{q^9} + \frac {1}{r^9} \right )^2 - 3\left ( \frac {1}{p^9 q^9 } + \frac {1}{p^9 r^9} + \frac {1}{q^9 r^9} \right ) \right )

1 p 27 + 1 q 27 + 1 r 27 = 7798252602 p 27 q 27 + p 27 r 27 + q 27 r 27 = 7798252602 \large \Rightarrow \frac {1}{p^{27}} + \frac {1}{q^{27}} + \frac {1}{r^{27}} = -7798252602 \Rightarrow p^{27} q^{27} + p^{27} r^{27} + q^{27} r^{27} = 7798252602

f 3 ( x ) = x 3 157957 x 2 + 7798252602 x 1 f_3 (x) = x^3 - 157957x^2 + 7798252602x - 1

Repeat one last time

p 81 + q 81 + r 81 3 ( p q r ) 27 = ( p 27 + q 27 + r 27 ) ( ( p 27 + q 27 + r 27 ) 2 3 ( p 27 q 27 + p 27 r 27 + q 27 r 27 ) ) p^{81} + q^{81} + r^{81} - 3(pqr)^{27} = (p^{27}+q^{27}+r^{27}) \left ( (p^{27}+q^{27}+r^{27})^2 - 3(p^{27} q^{27} +p^{27} r^{27} +q^{27} r^{27}) \right )

Modulo 1000 1000 :

p 81 + q 81 + r 81 157957 ( 15795 7 2 3 7798252602 ) 3 957 ( 95 7 2 3 602 ) 3 43 ( 4 3 2 1806 ) 3 148 \begin{aligned} p^{81} + q^{81} + r^{81} & \equiv & 157957 ( 157957^2 - 3 \cdot 7798252602 ) - 3 \\ & \equiv & 957 (957^2 - 3 \cdot 602 ) - 3 \\ & \equiv & -43 (43^2 - 1806 ) - 3 \\ & \equiv & \boxed{148} \\ \end{aligned}

isnt there a easy solution

alan alan - 7 years, 2 months ago

Log in to reply

no

Michael Mendrin - 7 years, 2 months ago

Log in to reply

I've presented a slightly easier solution, though the only difference is that I use algebra to treat the general case and iterate it, whereas Pi Han chose to labour through the calculations repeatedly.

Calvin Lin Staff - 7 years, 2 months ago

can we find - p 79 + q 79 + r 79 ? p^{79} + q^{79} + r^{79}?

U Z - 6 years, 5 months ago

Here's another potential approach, although its a lot more tedious than the approach above.

Let p n + q n + r n = S n p^n+q^n+r^n=S_n . Substituting p, q, r into the equation and summing, we have S 3 + 2 S 2 + 3 S 1 + 3 = 0 S_3+2 S_2+3 S_1+3=0 . Using Viete's relations, we have S 0 = 3 , S 1 = 2 , S 2 = 2 S_0=3,S_1=-2,S_2=-2 .

Multiplying through by p n , q n , r n p^n,q^n,r^n , and summing, we have the recursion S n + 3 + 2 S n + 2 + 3 S n + 1 + S n = 0 S_{n+3}+2 S_{n+2}+3 S_{n+1}+S_n=0 . Using the values above m o d 1000 mod 1000 we can chase down the value of S 81 S_{81} after a fair amount of tedious numerical work.

Karthik Tadinada - 7 years, 2 months ago
Calvin Lin Staff
Apr 11, 2014

This solution echos what Pi Han does, but presents it in a cleaner fashion. You will see glimpses of what he does. The main difference is that I use algebra to treat the general approach, and iterate it several times, as opposed to Pi Han who performs the calculations several times.


Lemma. Suppose that α , β , γ \alpha, \beta, \gamma are roots to the cubic equation x 3 + a x 2 + b x + c = 0 x^3 + ax^2 + bx + c = 0 . Then α 3 , β 3 , γ 3 \alpha^3, \beta^3, \gamma^3 are roots to the cubic equation X 3 + ( 3 c + a 3 3 a b ) X 2 + ( 3 c 2 + b 3 3 a b c ) X + c 3 = 0. X^3 +(3c +a^3 - 3ab) X^2 + (3c^2 + b^3 - 3abc)X + c^3 = 0.

Proof of Lemma. Apply Vieta's formula. Observe that
α 3 + β 3 + γ 3 = ( α + β + γ ) 3 3 ( α + β + γ ) ( α β + β γ + γ α ) + 3 α β γ \alpha^3 + \beta^3 + \gamma^3 = (\alpha + \beta + \gamma)^3 - 3 ( \alpha + \beta + \gamma) ( \alpha \beta + \beta \gamma + \gamma \alpha) + 3 \alpha \beta \gamma ,
α 3 β 3 + β 3 γ 3 + γ 3 α 3 = ( α β + β γ + γ α ) 3 3 ( α + β + γ ) ( α β + β γ + γ α ) ( α β γ ) + 3 ( α β γ ) 2 \alpha^3 \beta^3 + \beta^3 \gamma^3 + \gamma^3 \alpha^3 = (\alpha\beta + \beta \gamma + \gamma \alpha)^3 - 3 ( \alpha + \beta + \gamma) (\alpha\beta + \beta \gamma + \gamma \alpha) (\alpha \beta \gamma) + 3 ( \alpha \beta \gamma)^2 ,
α 3 β 3 γ 3 = ( α β γ ) 3 \alpha^3 \beta^3 \gamma^3 = ( \alpha \beta \gamma)^3 .


Note that Pi Han essentially used these formulas too.

Corollary. We thus get that

p 3 , q 3 , r 3 p^3, q^3, r^3 are roots of x 3 7 x 2 + 12 x + 1 = 0 , x^3 - 7x^2 + 12x + 1 = 0,

p 9 , q 9 , r 3 p^9, q^9, r^3 are roots of x 3 88 x 2 + 1983 x + 1 = 0 , x^3 - 88x^2 + 1983x + 1 = 0,

p 27 , q 27 , r 27 p^{27}, q^{27}, r^{27} are roots of x 3 157957 x 2 + 7798252602 + 1 = 0 , x^3 - 157957x^2 + 7798252602 + 1 = 0,

And finally, p 81 + q 81 + r 81 = ( 157957 ) 3 3 × 157957 × 7798252602 + 3 × 1 p^{81} + q^{81} + r^{81} = (157957)^3 - 3 \times 157957 \times 7798252602 + 3 \times 1 .


Alternative proof of lemma. (For those who don't like the 'amazing formulas' shown above. Though, this only works well for small degrees, since it is otherwise hard to force out the amazing formulas.)

Observe that x 3 + c = a x 2 b x x^3 + c = -ax^2 - bx . Cubing both sides, we obtain that

x 9 + 3 x 6 c + 3 x 3 c 2 + c 3 = a 3 x 6 3 a 2 b x 5 3 a b 2 x 4 b 3 x 3 . x^9 + 3x^6c + 3x^3c^2 +c^3 = - a^3 x^6 - 3a^2bx^5 - 3ab^2x^4 - b^3 x^3.

Every exponent of x x is a multiple of 3, with the exception of 3 a 2 b x 5 3 a b 2 x 4 - 3a^2bx^5 - 3ab^2x^4 . Luckily, we can express this as 3 a b x 3 ( a x 2 + b x ) = 3 a b x 3 ( x 3 c ) = 3 a b x 6 + 3 a b c x 3 -3abx^3 ( ax^2 + bx )= -3abx^3 ( - x^3 - c) = 3abx^6 + 3abcx^3 .

Hence, we have

x 9 + ( 3 c + a 3 3 a b ) x 6 + ( 3 c 2 + b 3 3 a b c ) x + c 3 = 0. x^9 + (3c +a^3 - 3ab) x^6 + (3c^2 + b^3 - 3abc)x + c^3 = 0.

As such, the cubic equation

X 3 + ( 3 c + a 3 3 a b ) X 2 + ( 3 c 2 + b 3 3 a b c ) X + c 3 = 0. X^3 +(3c +a^3 - 3ab) X^2 + (3c^2 + b^3 - 3abc)X + c^3 = 0.

would have roots of the form α 3 , β 3 , γ 3 \alpha^3 , \beta^3 , \gamma^3 .

Oh my goddddddd! So what if it is given a 5th degree polynomial and you want to find the sum of roots raised to the powers of power of 5? Can we generalize it to any positive integer n > 1 n>1 ?

Pi Han Goh - 7 years, 2 months ago

Log in to reply

Yes it does. The main difficulty is knowing how to express these (symmetric) polynomials in terms of the elementary symmetric polynomials. You can read about how any symmetric polynomial can be given by an expression involving only additions and multiplication of constants and elementary symmetric polynomials

Calvin Lin Staff - 7 years, 2 months ago

Well, I think I'm asking a little too much. Just stating α 7 β 7 + α 7 γ 7 + β 7 γ 7 \alpha^7 \beta^7 + \alpha^7 \gamma^7 + \beta^7 \gamma^7 in terms of α + β + γ , α β + α γ + β γ , α β γ \alpha + \beta + \gamma , \alpha \beta + \alpha \gamma + \beta \gamma, \alpha \beta \gamma is already such a hassle.

Pi Han Goh - 7 years, 2 months ago

Hah, I got lazy trying to state α 3 β 3 + α 3 γ 3 + β 3 γ 3 \alpha^3 \beta^3 + \alpha^3 \gamma^3 + \beta^3 \gamma^3 like how you did. So I chose the g n ( x ) = x 3 f n ( 1 x ) g_n (x) = x^3 f_n \left ( \frac {1}{x} \right ) route.

Pi Han Goh - 7 years, 2 months ago
Satyajit Mohanty
Sep 18, 2015

The main motive of this solution is to explain the fact that, not only p 81 + q 81 + r 81 p^{81} + q^{81} + r^{81} , but we can calculate the values of p 1 k + p 2 k + p 3 k + + p n k p_1^{k} + p_2^{k} + p_3^{k} + \ldots + p_n^k for any k 1 , k N k \geq 1, k \in \mathbb N using a single two step process.

This is a solution which uses Companion Matrix Methods and a heavy computational calculation.


Let P ( x ) = x 3 + 2 x 2 + 3 x + 1 P(x) = x^3+2x^2 + 3x+1 .

Let Q ( x ) Q(x) be a polynomial whose roots are 8 1 s t 81^{st} powers of the roots of P ( x ) P(x) .

We'll use the companion matrix of the polynomial P ( x ) P(x) namely:

A = ( 0 0 1 1 0 3 0 1 2 ) A = \begin{pmatrix} 0 & 0 & -1 \\ 1 & 0 & -3 \\ 0 & 1 & -2 \end{pmatrix}

for which the characteristic polynomial is det ( x I A ) = x 3 + 2 x 2 + 3 x + 1 \text{det}(xI-A) = x^3 + 2x^2 + 3x + 1 . For each integer k 1 k \geq 1 , the zeroes of the characteristic polynomial of A k A^{k} are the k t h k^{th} powers of the zeroes of P ( x ) P(x) . Taking this into account, we find that Q ( x ) = det ( x I A 81 ) Q(x) = \text{det}(xI-A^{81}) . Using Scientific Calculators, we obtain:

A 81 = ( 141006837910400 167222059610375 590313054904651 255868935683901 642673016741525 1603717105103578 167222059610375 590313054904651 537953093067777 ) A^{81} = \begin{pmatrix} 141006837910400 & 167222059610375 & -590313054904651 \\ 255868935683901 & 642673016741525 & -1603717105103578 \\ -167222059610375 & 590313054904651 & -537953093067777 \end{pmatrix}

Thus we have Q ( x ) Q(x) as:

Q ( x ) = det ( x I A 81 ) Q(x) = \text{det}(xI-A^{81})

= x 141006837910400 167222059610375 590313054904651 255868935683901 x 642673016741525 1603717105103578 167222059610375 590313054904651 x + 537953093067777 = \begin{vmatrix} x-141006837910400 & -167222059610375 & 590313054904651 \\ -255868935683901 & x-642673016741525 & 1603717105103578 \\ 167222059610375 & -590313054904651 & x+537953093067777 \end{vmatrix}

= x 3 245726761584148 x 2 + 474233136361262818167354353553 x + 1 = x^3 - 245726761584148x^2 + 474233136361262818167354353553x + 1

Thus, we know that if p , q , r p,q,r are the roots of P ( x ) P(x) , then p 81 , q 81 , r 81 p^{81}, q^{81}, r^{81} are the roots of Q ( x ) Q(x) .

Therefore, by Vieta's Formula, the sum of roots of Q ( x ) Q(x) or p 81 + q 81 + r 81 p^{81} + q^{81} + r^{81} equals 245726761584148 245726761584148 and it's last three digits being 148 148 .


Since I had no scientific calculators to perform heavy calculations, I preferred Wolfram-Mathematica.

  • I did this - Link 1 - to find the value of A 81 A^{81} .

  • I did this - Link 2 - to find the value of det ( x I A 81 ) \text{det}(xI-A^{81}) .

Moderator note:

You should learn how to diagonalize a matrix to find the exponentiation efficiently.

0 pending reports

×

Problem Loading...

Note Loading...

Set Loading...